0 Daumen
916 Aufrufe

Aufgabe:

Seien \( S_{1}=\left\{x \in \mathbb{R}^{n}:|x|=1\right\} \) und \( S_{2}=\left\{x \in \mathbb{R}^{n}:|x-a|=1\right\} \) mit \( a=(0,0, \ldots, 0,1) \) die Oberflächen zweier Kugeln im \( \mathbb{R}^{n}, n \geq 3 \).

a) Bestimmen Sie die Schnittmenge \( M=S_{1} \cap S_{2} \) und zeigen Sie, dass diese nicht leer und eine \( (n-2) \)-dimensionale \( C^{\infty} \)-Mannigfaltigkeit in \( \mathbb{R}^{n} \) ist.

b) Durch Auflösen von

\( f\left(x_{1}, \ldots, x_{n}\right)=\left(\begin{array}{c} |x|-1 \\ |x-a|-1 \end{array}\right)=\left(\begin{array}{l} 0 \\ 0 \end{array}\right) \)

nach zwei Variablen \( x_{i}, x_{j} \) erhält man eine Parameterdarstellung der Fläche. Nach welchen Variablen kann man in einer Umgebung eines Punktes \( x^{*} \in M \) auflösen, was sind dann die Parameter?


Ansatz/Problem:

Schnittmengen habe ich bestimmt, weiß aber nicht, ob es stimmt:

Bild Mathematik

Avatar von

1 Antwort

0 Daumen

Warum nur für den Fall n=3? Deine Umformungen sehen auch nicht zielführend aus

x3 = -1 ist ganz falsch.


Ansatz schon zu erkennen (die Gleichstellung). Aus dieser geht hervor, dass

$$ x_n^2 = (x_n-1)^2 $$

gelten muss. Welchen Wert kann \(x_n\) also nur haben?

Avatar von 23 k
Okay!
Naja aber wenn :
xn2 =( xn -1)2 gilt dann hat das doch für kein xeine Lösung oder denk ich gerade verkehrt ?
Denn es gilt doch wenn ich auf beiden Seiten Wurzel ziehe
xn = xn - 1
und wenn ich dann xn  auf eine Seite bringen und dann hätten wir doch das 0 =-1 .
Und wir hatten gedacht machen das für n =3  und zeigen das es für n gilt per Induktion.
LG

Hi änder bitte die Antwort zu nem Kommentar, macht ja kein Sinn das hier als Antwort stehen zu lassen.

Ah ok durchs Wurzelziehen hast du dir selber ein Bein gestellt. Versuchs mal anders

-> Multiplizier die rechte Seite aus und dann löse nach \(x_n\) auf.

Beim Wurzelziehen muss man aufpassen da es auch negative Lösungen geben kann ;)

Induktion brauchst du hier nicht halt es von Anfang an allgemein.

Ah okay dankeschön stimmt.

Naja dann gilt es ja für Okay!
Naja aber wenn :
xn =1/2

Ja und jetzt kannst du ja deine Schnittmenge angeben. Um zu zeigen dass es sich um eine Mannigfk handelt kannst du bspw. den Satz vom regulären Wert benutzen oder auch direkt argumentieren (Falls ihr Kugeloberflächen als Mannigfaltigkeiten schon ausreichend behandelt habt) wenn dir die Schnittmenge bekannt vorkommt.

Stimmt das denn soweit ,mit der Schnittmenge.

Wir haben weder  den Satz nach Kugelflächen ausreichend  behandelt.

Lg

Bild Mathematik

Die letzte Zeil ist falsch, M ist nicht die Menge aller Vektoren mit dem letzten Eintrag 1/2. M ist die Menge aller Vektoren aus S1 bzw. S2 mit letztem Eintrag 1/2. Da ich nicht weiß was ihr bisher gemacht habt musst du wohl selber in dein Skript und Übungsmitschrift schauen was ihr bisher gemacht habt um Mannigfaltigkeiten nachzuweisen.

Ein anderes Problem?

Stell deine Frage

Willkommen bei der Mathelounge! Stell deine Frage einfach und kostenlos

x
Made by a lovely community